GH from MO's user avatar
GH from MO's user avatar
GH from MO's user avatar
GH from MO
  • Member for 13 years, 3 months
  • Last seen this week
18 votes

Resubmitting a paper

18 votes

A quadratic form represents all primes except for the primes 2 and 11.

18 votes

Primitive roots

18 votes
Accepted

A partial converse to Bertrand's Postulate

18 votes
Accepted

Is there always, for a given prime $p$, a prime $\ell<p$ that is not a quadratic residue mod $p$?

18 votes

What is the high-concept explanation on why real numbers are useful in number theory?

17 votes

What is the state of our ignorance about the normality of pi?

17 votes

Why are lacunary series so badly behaved?

17 votes

primorial puzzlement

17 votes
Accepted

Does a function exist which is not Riemann integrable and satisfies the given condition:

17 votes
Accepted

Is $\limsup_{x\to\infty}\big(\sum\limits_{d|3^x-1}{1/d}\big)/\big(\sum\limits_{p<x}1/p\big)<\infty$?

17 votes
Accepted

Is there always a real $x$ such that $\cos n_1 x + \cos n_2 x + \cos n_3 x < -2$?

17 votes
Accepted

Modular forms with finitely many or very few non-zero Fourier coefficients

17 votes

Collection of equivalent forms of Riemann Hypothesis

16 votes
Accepted

Origin of Hecke operators

16 votes
Accepted

A recursive formula

16 votes
Accepted

simple conjecture on palindromes in base 10

16 votes
Accepted

Optimality of the Riemann Hypothesis

16 votes

Converse to Erdős' conjecture on arithmetic progressions

16 votes
Accepted

binomial coefficients are integers because numerator and denominator form pairs?

16 votes
Accepted

Euler's proof of $\frac{\pi}{6}=1-\frac{1}{2}-\frac{1}{3}+\frac{1}{4}+\frac{1}{5}+\frac{1}{6}-\cdots$

16 votes
Accepted

Proving Mertens' theorem using the prime number theorem

16 votes

Is $\lceil \frac{n}{\sqrt{3}} \rceil > \frac{n^2}{\sqrt{3n^2-5}}$ for all $n > 1$?

16 votes
Accepted

Which of these sums appear most often?

16 votes
Accepted

How to prove that $\int _0^\infty\frac{\text{arcsinh}^nx}{x^m}dx$ is a rational combination of zeta values?

16 votes
Accepted

How to prove a known inequality from a book

16 votes

Is the Euler product formula always divergent for 0<Re(s)<1?

16 votes

Series whose convergence is not known

16 votes
Accepted

Is there a constant $c>0$, such that every natural number $n>1$ is the sum of primes, each with size at least $cn$?

16 votes

About a reparable error discovered in a submitted article

1 2 3
4
5
34